LSAT and Law School Admissions Forum

Get expert LSAT preparation and law school admissions advice from PowerScore Test Preparation.

 Administrator
PowerScore Staff
  • PowerScore Staff
  • Posts: 8917
  • Joined: Feb 02, 2011
|
#24423
Complete Question Explanation

Weaken-CE. The correct answer choice is (E)

Since hard water contains more magnesium than soft water and people suffering from heart disease tend to have lower levels of magnesium in their blood, the author concludes that drinking soft water puts people at an increased risk of heart disease. Implicit in this conclusion is the assumption that the lower levels of magnesium are the cause, and not the effect, of heart disease:
  • Cause ..... ..... ..... ..... ..... ..... Effect
    Low levels of magnesium ..... :arrow: ..... Heart disease
To weaken this argument, it is logical to hypothesize that heart disease is not caused by low levels of magnesium, but rather that a third cause is responsible for both the low levels of magnesium and the heart disease, or else that heart disease somehow causes the body to retain less magnesium. Answer choice (E) is therefore correct.

Answer choice (A): That magnesium deficiency is common plays no role in this argument, and certainly does not weaken the causality implicit in the author’s conclusion. This answer choice is incorrect.

Answer choice (B): If magnesium is needed to prevent sodium from increasing blood presure, low levels of magnesium might fail to avert the ill effects of sodium and therefore lead to the development of heart disease and high blood pressure. Since this answer choice strengthens the cause-and-effect relationship between low levels of magnesium and heart disease, it does the exact opposite of what is needed and is therefore incorrect.

Answer choice (C): This answer choice merely suggests another reason why magnesium intake might be disrupted. There is no evidence that the decreased ability to metabolize magnesium leads to decreased magnesium intake (for all we know, it might lead to a higher concentration of magnesium in the blood). Furthermore, there is no indication that the elderly are at a higher risk of developing heart disease and high blood pressure. This answer choice is incorrect.

Answer choice (D): Even if magnesium supplements inhibited the effectiveness of many medicines used to treat heart disease, this does not weaken the author’s conclusion that the low levels of magnesium can contribute to the development of heart disease too. It is perfectly possible that any level of magnesium intake that falls outside the normal range (too low or too high) can adversely affect cardiac health. This answer choice is incorrect.

Answer choice (E): This is the correct answer choice. See discussion above.
 lathlee
  • Posts: 652
  • Joined: Apr 01, 2016
|
#42714
Hi. I know why E is correct but can you explain in more easier terms why C) is incorrect cuz C) technically introduces alternative reason for Heart disease
 Adam Tyson
PowerScore Staff
  • PowerScore Staff
  • Posts: 5153
  • Joined: Apr 14, 2011
|
#42730
Actually, lathlee, answer C says nothing about heart disease! It doesn't do anything to hurt the claim that those who drink soft water, with less magnesium, are at higher risk for heart disease. Are you bringing in outside information that as people age, their risk of heart disease increases? Careful! If the stimulus doesn't say it, it's not something you should assume!
 theamazingrace
  • Posts: 59
  • Joined: Oct 17, 2020
|
#83025
I eliminated C because when I saw the word metabolize I thought the process of breaking down something. So, there could still be high levels of magnesium in the bloodstream of older people that are just not being broken down. This is why I thought it did not hurt the argument. Is this a good reason to eliminate C?

Thank you in advance!
 Robert Carroll
PowerScore Staff
  • PowerScore Staff
  • Posts: 1787
  • Joined: Dec 06, 2013
|
#84291
grace,

I don't even know how heart attack and other health risks correlate with age, because the stimulus doesn't talk about that. I'm also not sure what effect decreased metabolization of magnesium would have on blood levels - up, because it's not broken down? Maybe down, because it's not being metabolized in some other area? Or what? The relevance of this to the stimulus is up in the air.

Robert Carroll
 lsat_student0543
  • Posts: 7
  • Joined: Feb 06, 2021
|
#85818
I chose E but I can’t shake D. Doesn’t it weaken the idea that low levels of magnesium contribute to heart disease, or does D require the assumption that someone who take the magnesium supplements takes enough of them to rise above the level considered “low”?

Administrator wrote: Wed May 11, 2016 3:21 pm Complete Question Explanation

Weaken-CE. The correct answer choice is (E)

Since hard water contains more magnesium than soft water and people suffering from heart disease tend to have lower levels of magnesium in their blood, the author concludes that drinking soft water puts people at an increased risk of heart disease. Implicit in this conclusion is the assumption that the lower levels of magnesium are the cause, and not the effect, of heart disease:
  • Cause ..... ..... ..... ..... ..... ..... Effect
    Low levels of magnesium ..... :arrow: ..... Heart disease
To weaken this argument, it is logical to hypothesize that heart disease is not caused by low levels of magnesium, but rather that a third cause is responsible for both the low levels of magnesium and the heart disease, or else that heart disease somehow causes the body to retain less magnesium. Answer choice (E) is therefore correct.

Answer choice (A): That magnesium deficiency is common plays no role in this argument, and certainly does not weaken the causality implicit in the author’s conclusion. This answer choice is incorrect.

Answer choice (B): If magnesium is needed to prevent sodium from increasing blood presure, low levels of magnesium might fail to avert the ill effects of sodium and therefore lead to the development of heart disease and high blood pressure. Since this answer choice strengthens the cause-and-effect relationship between low levels of magnesium and heart disease, it does the exact opposite of what is needed and is therefore incorrect.

Answer choice (C): This answer choice merely suggests another reason why magnesium intake might be disrupted. There is no evidence that the decreased ability to metabolize magnesium leads to decreased magnesium intake (for all we know, it might lead to a higher concentration of magnesium in the blood). Furthermore, there is no indication that the elderly are at a higher risk of developing heart disease and high blood pressure. This answer choice is incorrect.

Answer choice (D): Even if magnesium supplements inhibited the effectiveness of many medicines used to treat heart disease, this does not weaken the author’s conclusion that the low levels of magnesium can contribute to the development of heart disease too. It is perfectly possible that any level of magnesium intake that falls outside the normal range (too low or too high) can adversely affect cardiac health. This answer choice is incorrect.

Answer choice (E): This is the correct answer choice. See discussion above.
User avatar
 KelseyWoods
PowerScore Staff
  • PowerScore Staff
  • Posts: 1079
  • Joined: Jun 26, 2013
|
#85917
Hi lsat_student0543!

Answer choice (D) states: "The ingestion of magnesium supplements inhibits the effectiveness of many medicines used to treat high blood pressure and heart disease." From the stimulus, we know that "people being treated for these conditions tend to have lower levels of magnesium in their blood" and the author seems to assume that the low magnesium is causing these heart conditions because the author concludes that those drinking water that is lower in magnesium have an increased risk of these conditions. Even if taking magnesium supplements inhibits the effectiveness of the medicines used to treat these diseases, that doesn't tell us anything about whether low magnesium was an original cause for these diseases. The magnesium supplements could interact with ingredients in the medications being used to treat people who have already been diagnosed with those heart conditions, whether or not low magnesium levels initially caused the heart conditions.

Hope this helps!

Best,
Kelsey
 lsat_student0543
  • Posts: 7
  • Joined: Feb 06, 2021
|
#86269
Kelsey - thank you. yea, that helped me.
KelseyWoods wrote: Sat Mar 27, 2021 9:35 am Hi lsat_student0543!

Answer choice (D) states: "The ingestion of magnesium supplements inhibits the effectiveness of many medicines used to treat high blood pressure and heart disease." From the stimulus, we know that "people being treated for these conditions tend to have lower levels of magnesium in their blood" and the author seems to assume that the low magnesium is causing these heart conditions because the author concludes that those drinking water that is lower in magnesium have an increased risk of these conditions. Even if taking magnesium supplements inhibits the effectiveness of the medicines used to treat these diseases, that doesn't tell us anything about whether low magnesium was an original cause for these diseases. The magnesium supplements could interact with ingredients in the medications being used to treat people who have already been diagnosed with those heart conditions, whether or not low magnesium levels initially caused the heart conditions.

Hope this helps!

Best,
Kelsey

Get the most out of your LSAT Prep Plus subscription.

Analyze and track your performance with our Testing and Analytics Package.